November 10, 2009

Homework #25 Answer

If someone puts too much pressure on the bottom shelf of a bookcase, the entire bookcase will collapse. Shelly's bookcase has collapsed. Therefore, someone must have put too much pressure on the bottom shelf.

Which one of the following criticisms best describes a weakness in the argument above?

A. It supports its conclusion with irrelevant evidence.

(A) is incorrect because the evidence, while inadequate to support the conclusion, is nonetheless relevant. The evidence here is that the bookcase collapsed, and that putting too much pressure on the bottom shelf of any bookcase will cause that bookcase to collapse. The conclusion is that the collapse of Shelly's bookcase must have happened because someone put too much pressure on the bottom shelf. The evidence is relevant because it makes the conclusion more likely to be true; if the bookcase collapsed, and too much pressure on the bottom shelf can cause it, then the collapse of the bookcase could suggest that someone put too much pressure on the bottom shelf. The problem is that the evidence does not by itself prove that the conclusion must be true.

B. It changes the meaning of the word "collapse" from "partial collapse" to "full collapse."

(B) is incorrect because the argument does not hinge on the definition of "collapse." This is a simple reverse cause-and-effect argument; it doesn't matter whether the collapse was "partial" or "full," or whether the author meant "partial" or "full" when he wrote the word "collapse." At issue is the cause of the collapse, not the extent of it.

C. It concludes that an outcome must have been caused by one particular factor, which may be only one of many possible causes of that outcome.

(C) is correct. As discussed above, the flaw in the argument is that the author has assumed the cause based only on the outcome and nothing more. This classic logical fallacy came up in Homework #23 and #24 as well. Just because X can cause Y, or has a tendency to cause Y, does not mean that if Y occurs it must (or could only) have been caused by X. Only if we know that there is no other possible cause can we assume the cause from the outcome alone.

D. It overlooks the possibility that some bookshelves have only one shelf.

(D) is incorrect because "the possibility that some bookshelves have only one shelf" does not make it more or less likely that the collapse of this particular bookcase was caused by pressure on the bottom shelf. Therefore the fact that the possibility was "overlooked" is not a weakness, because there is nothing wrong with ignoring irrelevant factors when making an argument. Indeed, ignoring irrelevancies actually strengthens the argument.

E. It draws an overly broad conclusion from small, insignificant pieces of evidence.

(E) is incorrect because the conclusion is not "overly broad;" it is in fact overly narrow, assuming only one of many possible causes for a specific outcome. Neither is the evidence (i.e., the collapse of the bookcase) "small" or "insignificant."